ChaseDream
搜索
返回列表 发新帖
查看: 3012|回复: 6
打印 上一主题 下一主题

[原创]OG10逻辑分类剖析

[复制链接]
楼主
发表于 2007-8-25 15:58:00 | 只看该作者

[原创]OG10逻辑分类剖析

Assumption

8. To prevent some conflicts of interest, Congress could prohibit high-level government officials from accepting positions as lobbyists for three years after such officials leave government service. One such official concluded, however, that such a prohibition would be unfortunate because it would prevent high-level government officials from earning a livelihood for three years.

 

The official’s conclusion logically depends on which of the following assumptions?

A. Laws should not restrict the behavior of former government officials.

B. Lobbyists are typically people who have previously been high-level government officials.

C. Low-level government officials do not often become lobbyists when they leave government service.

D. High-level government officials who leave government service are capable of earning a livelihood only as lobbyists.
            
D

E. High-level government officials who leave government service are currently permitted to act as lobbyists for only three years.

 

8.

The official argues that prohibiting high-level government officials from accepting positions as lobbyists for three years would prevent the officials from earning a livelihood for that period. The reasoning tacitly excludes the possibility of such officials earning a living through work other than lobbying. Therefore, D, which expresses this tacit assumption, is the best answer.

 

The official’s argument does not depend on the assumption in A, since the argument would not be invalidated if some restrictions on the behavior of government officials were desirable. The official’s argument does not depend on the assumption in B, since the argument would not be invalidated if lobbyists were not typically former high-level government officials. The official’s argument does not depend on the assumption in C, since the argument would not be invalidated if former low-level government officials did often become lobbyists. The official’s argument does not depend on the assumption in E, since the argument would not be invalidated if former high-level government officials could act as lobbyists indefinitely(不确定地).

Gap弥补

 

 

21. When limitations were in effect on nuclear-arms testing, people tended to save more of their money, but when nuclear-arms testing increased, people tended to spend more of their money. The perceived threat of nuclear catastrophe, therefore, decreases the willingness of people to postpone consumption for the sake of saving money.

 

The argument above assumes that

A. the perceived threat of nuclear catastrophe has increased over the years.

B. most people supported the development of nuclear arms

C. people’s perception of the threat of nuclear catastrophe depends on the amount of nuclear-arms testing being done

D. the people who saved the most money when nuclear-arms testing was limited were the ones who supported such limitations C

E. there are more consumer goods available when nuclear-arms testing increases

 

21.

On the basis of an observed correlation between arms testing and people’s tendency to save money, the argument concludes that there is a causal connection between a perception of threat and the tendency not to save. That connection cannot be made unless C, linking the perception of threat to the amount of testing being done, is assumed to be true. Therefore, C is the best answer.

 

The conclusion does not depend on there having been an increase in the perceived threat over time or on how many people supported the development of nuclear arms. Hence, neither of A and B is assumed. The argument does not deal with those who supported arms limitations or with the availability of consumer goods. Thus, D and E are not assumed.

A==B

快速确定A和B,然后确定逻辑推理过程中缺失的潜在因素,并加以弥补

 

36. If the airspace around centrally located airports were restricted to commercial airliners and only those private planes equipped with radar, most of the private-plane traffic would be forced to sue outlying airfields. Such a reduction in the amount of private-plane traffic would reduce the risk of midair collision around the centrally located airports.

 

The conclusion draw in the first sentence depends on which of the following assumptions?

A. Outlying airfields would be as convenient as centrally located airports for most pilots of private planes.

B. Most outlying airfields are not equipped to handle commercial-airline traffic.

C. Most private planes that use centrally located airports are not equipped with radar.

D. Commercial airliners are at greater risk of becoming involved in midair collisions than are private planes. C

E. A reduction in the risk of midair collision would eventually lead to increases in commercial-airline traffic.

Aè只允许商业航线以及安装雷达的私人飞机..”

Bè很多私人飞机将被迫迁往偏僻的机场

     è C是很多私人飞机没有安装雷达

36.

The first sentence concludes that prohibiting private planes that are not radar-equipped from centrally located airports would force most private planes away from those airports. This conclusion cannot be true unless it is true that, as C says, most private planes that use these airports are not radar-equipped. Therefore, the first sentence’s conclusion assumes this choice, which is thus the best answer.

 

The conclusion need not assume that outlying airfields are convenient for private planes (Choice A), since the restrictions would give planes that are not radar equipped no choice. The conclusion concerns only how the radar requirement would affect the volume of private plane traffic, so B, D and E, which deal with commercial planes and with risk of midair collision, need not be assumed.

沙发
 楼主| 发表于 2007-8-25 15:59:00 | 只看该作者

继续

 

48. A researcher discovered that people who have low levels of immune-system activity tend to score much lower on tests of mental health than do people with normal or high immune-system activity. The researcher concluded from this experiment that the immune system protects against mental illness as well as against physical disease

.

The researcher’s conclusion depends on which of the following assumptions?

A. High immune-system activity protects against mental illness better than normal immune-system activity does.[未提及的比较]

B. Mental illness is similar to physical disease in its effects on body systems.[无关选项]

C. People with high immune-system activity cannot develop mental illness. [未提及的内容]

D. Mental illness does not cause people’s immune-system activity to decrease. D

E. Psychological treatment of mental illness is not as effective as is medical treatment. [未提及的内容]
        

Aè研究发现..低免疫系统活性的人,在精神测试方面得分低

Bè免疫系统不仅仅避免生理疾病亦能避免精神疾病

     è
        
免疫系统疾病本身不会导致免疫系统活性低

[前提为A,结论为B,假设说(A) 不成立]

 

 

48.

The researcher concludes from the association of low immune-system activity with low mental-health sores that, in effect, immune system activity can inhibit mental illness. If, contrary to D, mental illness can depress immune-system activity, the association mentioned does not support the researcher’s conclusion. So D must be assumed.

 

Normal immune-system activity could protect against mental illness without high-immune system activity offering increased protection or prevention, contrary to what A and C state, so neither of A and C is assumed. The conclusion does not depend on there being a similarity between mental and physical illness or a difference in treatments, so B and E are not assumed.

 

63. Traditionally, decision-making by managers that is reasoned step-by-step has been considered preferable to intuitive decision-making. However, a recent study found that top managers used intuition significantly more than did most middle-or lower-level managers. This confirms the alternative view that intuition is actually more effective than careful, methodical reasoning.

Aè研究表明高层更多使用直觉

Bè所以直觉比SyS更有效率

  C è高层比很多中层更有效率

 

 

The conclusion above is based on which of the following assumptions?

(A) Methodical, step-by-step reasoning is inappropriate for making many real-life management decisions.

(B) Top managers have the ability to use either intuitive reasoning or methodical, step-by-step reasoning in making decisions.

(C) The decisions made by middle-and lower-level managers can be made as easily by using methodical reasoning as by using intuitive reasoning.

(D) Top managers use intuitive reasoning in making the majority of their decisions.
            
E

(E) Top managers are more effective at decision-making than middle-or lower-level managers
            

 

63.

If top managers are not the more effective decision makers, then the fact that they use intuition more often than lower-level managers does not support the conclusion that intuition is more effective. Because the argument must assume E, choice E is the best answer.

 

To the extent that less effective methods are inappropriate, the passage does not assume A, but argues for it. Since the argument leaves open the possibility of situations in which top managers are unable to use one of the methods, choice B is inappropriate. Since the ease with which a method is implemented is not at issue, choice C is inappropriate. The argument is consistent with managers at all levels using intuition in the minority of decisions made. Thus, choice D is inappropriate.

 

76. A famous singer recently won a lawsuit against an advertising firm for using another singer in a commercial to evoke the famous singer’s well-known rendition of a certain song. As a result of the lawsuit, advertising firms will stop using imitators in commercials. Therefore, advertising costs will rise, since famous singers’ services cost more than those of their imitators.

Aè著名歌手赢得歌曲版权官司

Bè广告成本上升

    Cè广告都需要使用著名歌手的歌曲

 

The conclusion above is based on which of the following assumptions?

(A) Most people are unable to distinguish a famous singer’s rendition of a song from a good imitator’s rendition of the same song.

(B) Commercials using famous singers are usually more effective than commercials using imitators of famous singers.

(C) The original versions of some well-known songs are unavailable for use in commercials.

(D) Advertising firms will continue to use imitators to mimic the physical mannerisms of famous singers.
            
E

(E) The advertising industry will use well-known renditions of songs in commercials.
                

 

76.

If choice E were not assumed, the costs of the services of the famous singers of well-known renditions of songs would not be said to affect advertising costs. Since advertising costs are, however, projected to rise because of the relatively high cost of famous singers’ services, choice E is assumed and is the best answer.

 

Choice A is irrelevant to the argument, since famous singers’ service cost more than imitators’ anyway. The argument addresses commercials’ cost, not their effectiveness, so choice B is not assumed. The argument assumes that some well-known renditions of songs are available, but does not require that any versions be unavailable (choice C). Since the argument states that advertising firms will stop using imitators, choice D is not assumed.

 

80.The program to control the entry of illegal drugs into the country was a failure in 1987. If the program had been successful, the wholesale price of most illegal drugs would not have dropped substantially in 1987.

Aè如果非法药物控制成功

Bè非法药物价格不会急剧下跌

Cè排除它因:否定药物价格下跌是由于其它原因,例如:需求下滑

[前提为A,结论为B,假设排除它因]

 

 

The argument in the passage depends on which of the following assumptions?

(A) The supply of illegal drugs dropped substantially in 1987.

(B) The price paid for most illegal drugs by the average consumer did not drop substantially in 1987.

(C) Domestic production of illegal drugs increased at a higher rate than did the entry of such drugs into the country.

(D) The wholesale price of a few illegal drugs increased substantially in 1987. E

(E) A drop in demand for most illegal drugs in 1987 was not the sole cause of the drop in their wholesale price.

 

80.

The only choice that must be true in order to conclude legitimately from the drop in the wholesale price of illegal drugs that the program was a failure is choice E, the best answer. If the drop in price was caused by a drop in demand, there is no reason to suspect that there has been any increase in supply caused by drugs entering the country.

 

The other choices can be false without affecting the argument. The supply of illegal drugs need not have dropped (choice A), and the retail price could have dropped (choice B). The entry of illegal drugs could have risen at a higher rate than domestic production (choice C), and no illegal drug need have undergone a substantial price rise (choice D).

板凳
 楼主| 发表于 2007-8-25 16:00:00 | 只看该作者

继续

 

103. The technological conservatism of bicycle manufacturers is a reflection of the kinds of demand they are trying to meet. The only cyclists seriously interested in innovation and willing to pay for it are bicycle racers. Therefore, innovation in bicycle technology is limited by what authorities will accept as standard for purpose of competition in bicycle races.

 

Which of the following is an assumption made in drawing the conclusion above?

(A) The market for cheap, traditional bicycles cannot expand unless the market for high-performance competition bicycles expands.

(B) High-performance bicycles are likely to be improved more as a result of technological innovations developed in small workshops than as a result of technological innovations developed in major manufacturing concerns.

(C) Bicycle racers do not generate a strong demand for innovations that fall outside what is officially recognized as standard for purposes of competition.

(D) The technological conservatism of bicycle manufacturers results primarily from their desire to manufacture a product that can be sold without being altered to suit different national markets. C

(E) The authorities who set standards for high-performance bicycle racing do not keep informed about innovative bicycle design.
            

Aè自行车的技术革新是需求的反映

Bè因此,自行车的技术革新由自行车比赛的限定决定

Cè比赛运动员仅仅对符合比赛要求的革新感兴趣

[前提为A,结论为BGAP弥补]

 

103.

If racers, the only cyclists interested in innovation, created a strong demand for innovations for purposes other than official competition, then the conclusion would not follow. Therefore choice C-which asserts that racers generate no such demand-is assumed and is the best answer.

 

Since the argument is stated generally in terms of where demand for innovation lies and how manufacturers respond to demand, no assumption is made about the structure of the market for bicycles themselves (choice A) nor about which manufactures are most likely to produce innovations (choice B). Choice D presents another pressure toward technological conservatism, but the pressure is not required by the argument. Finally, the authorities may keep a close eye on innovation (choice E) without the argument being affected.

 

104. Spending on research and development by United States businesses for 1984 showed an increase of about 8 percent over the 1983 level. This increase actually continued a downward trend evident since 1981 – when outlays for research and development increased 16.4 percent over 1980 spending. Clearly, the 25 percent tax credit enacted by Congress in 1981, which was intended to promote spending on research and development, did little or nothing to stimulate such spending.

Aè “1984年比1983年增长8%19811980增长16.4%”

Bè国会1981年的措施起到很少的作用或者根本不起作用

Cè没有1981年的国会的措施的话,该增长也不会更低如果该假设不成立,那么结论也不会成立

[前提为A,结论为B,假设是结论的另外一种说法]

 

 

The conclusion of the argument above cannot be true unless which of the following is true?

(A) Business spending on research and development is usually directly proportional to business profits.[无关选项]

(B) Business spending for research and development in 1985 could not increase by more than 8.3%.[无关选项]

(C) Had the 1981 tax credit been set higher than 25%, business spending for research and development after 1981 would have increased more than it did. [无关选项]

(D) In the absence of the 25% tax credit, business spending for research and development after 1981 would not have been substantially lower than it was.
            
D[说明即使国会不实施该政策,该spending也不会更低== 说明没有太大影响,进而说明该措施失败]

(E) Tax credits market for specific investments are rarely effective in inducing businesses to make those investments. [无关选项]

 

104.

The conclusion that the tax credit did nothing to stimulate spending on research and development would not be true if, without the credit, such spending would have been even lower than it actually was. Thus choice D must be true for the conclusion to be true and is the best answer.

 

Since a tax credit generally improves business profits, if the conclusion is true choice A is unlikely to be true. If the tax credit was ineffective, some other factors must determine the level of spending, and could lead to much higher levels of spending in 1985 (against choice B), and could render a higher level of tax credit ineffective (against choice C), but it could be that credits are generally effective (against choice E).

 

122. When people evade income taxes by not declaring taxable income, a vicious cycle results. Tax evasion forces lawmakers to raise income tax rates, which causes the tax burden on nonevading taxpayers to become heavier. This, in turn, encourages even more taxpayers to evade income taxes by hiding taxable income.

Aè逃税迫使提高税率

Bè导致更多人逃税è
        
因此导致一个恶性循环

Cè
        
解释逃税为何必然导致税率提高的原因è税率制定时没有考虑到逃税带来的利润的损失

[前提为A,结论为B,假定加强了前提的必然性,对原因进行保护也是Assumption的一种]

 

The vicious cycle described above could not result unless which of the following were true?

(A) An increase in tax rates tends to function as an incentive for taxpayers to try to increase their pretax incomes.[无关选项]

(B) Some methods for detecting tax evaders, and thus recovering some tax revenue lost through evasion, bring in more than they cost, but their success rate varies from years to year. .[无关选项]

(C) When lawmakers establish income tax rates in order to generate a certain level of revenue, they do not allow adequately for revenue that will be lost through evasion.

(D) No one who routinely hides some taxable income can be induced by a lowering of tax rates to stop hiding such income unless fines for evaders are raised at the same time.
            
C

(E) Taxpayers do not differ from each other with respect to the rate of taxation that will cause them to evade taxes.
            
.[
无关选项]

 

122.

For tax evasion to force a raise in income tax rates it must be true that tax evasion causes actual tax revenues to fall short of revenue needs. This is the situation that choice C describes; choice C is therefore the best answer.

 

None of the other choices states a requirement for the vicious cycle to result. Increasing in pretax incomes (income A) would tend to work against perpetuation of the cycle. Success at catching tax evaders (choice B) should likewise have an inhibiting effect. Choice D describes how problems in breaking existing habits of tax evasion might be overcome. Choice E essentially denies that raising the tax rate in response to some tax evasion could cause additional tax-payers to evade taxes.

地板
 楼主| 发表于 2007-8-25 16:00:00 | 只看该作者

继续

 

 

123. Advertisement: Today’s customers expect high quality. Every advance in the quality of manufactured products raises customer expectations. The company that is satisfied with the current quality of its products will soon find that its customers are not. At MegaCorp, meeting or exceeding customer expectations is our goal.

Aè顾客要求高质量,并且其期望值不断提高

Bè “MC公司的目标是不断满足或超越顾客的质量期望

Cè结论的实施=〉质量的不断提高是实际可行的

[前提为A,结论为B,假定对结论的切实可行性加以保护]

 

Which of the following must be true on the basis of the statements in the advertisement above?

 

(A) MegaCorp’s competitors will succeed in attracting customers only if those competitors adopt MegaCorp’s goal as their own.

(B) A company that does not correctly anticipate the expectations of its customers is certain to fail in advancing the quality of its products.

(C) MegaCorp’s goal is possible to meet only if continuing advances in product quality are possible. [如果C不能被满足,则结论必然不成立]

(D) If a company becomes satisfied with the quality of its products, then the quality of its products is sure to decline. C

(E) MegaCorp’s customers are currently satisfied with the quality of its products.

 

123.

MegaCorp wishes to at least meet customer expectations. Since these expectations will always tend to move beyond whatever level of quality MegaCorp happens to have attained, MegaCorp will, as choice C indicates, be able to meet its goal only if continuing improvements in the quality of its products are possible. Choice C is thus the best answer.

 

Choice A is incorrect since success in attracting customers depends only on actual product quality, not on a company’s goals regarding quality. Since quality improvements can themselves shape customer expectations, choice B is incorrect. Since nothing has been said to indicate a difficulty with maintaining a given level of product quality, choice D is incorrect. Since having a goal does not imply meeting it, choice E is incorrect.

 

 

130. Bank depositors in the United States are all financially protected against bank failure because the government insures all individuals' bank deposits. An economist argues that this insurance is partly responsible for the high rate of bank failures, since it removes from depositors any financial incentive to find out whether the bank that holds their money is secure against failure. If depositors were more selective, then banks would need to be secure in order to compete for depositors' money.

Aè如果储户对银行选择

Bè银行将更关注安全以竞争储户的资金

Cè储户懂得如何选择银行银行竞争储户资金不会导致不安全

[前提为A,结论为B,假定对前提的可行性进行保护]

 

The economist's argument makes which of the following assumptions?

(A) Bank failures are caused when big borrowers default on loan repayments.

(B) A significant proportion of depositors maintain accounts at several different banks.

(C) The more a depositor has to deposit, the more careful he or she tends to be in selecting a bank.

(D) The difference in the interest rates paid to deposit
            
ors by different banks is not a significant factor in bank failures.
            
E

(E) Potential depositors are able to determine which banks are secure against failure.

 

130.

Giving potential depositors a financial incentive to select only secure banks will not lead to increased bank security unless the potential depositors can distinguish banks that actually are secure from those that are not. Choice E is a statement of this prerequisite and is thus the best answer.

 

The argument is about choosing or avoiding banks likely to fail, regardless of how the failure comes about, so neither choice A nor choice D is specifically assumed. The argument is consistent with each depositor’s money being held by a single bank, so B is not assumed. The argument neither asserts nor assumes that depositors currently exercise care in selecting the banks where they deposit their money. Therefore choice C, in particular, is not assumed.

 

139. Roland: The alarming fact is that 90 percent of the people in this country now report that they know someone who is unemployed.☆☆☆☆

 

Sharon: But a normal, moderate level of unemployment is 5 percent, with 1 out of 20 workers unemployed. So at any given time if a person knows approximately 50 workers, 1 or more will very likely be unemployed.

Aè失业的正常水平是5%”

Bè所以,一个人如果认识50个工人,那么1个或更多可能是失业人员

Cè失业的人群通常不会集中在一个地区

[前提为A,结论为B,假定对结论进行解释]

 

Sharon's argument relies on the assumption that

(A) normal levels of unemployment are rarely exceeded[无关选项]

(B) unemployment is not normally concentrated in geographically isolated segments of the population[失业通常不是集中在地理隔绝的区域]

(C) the number of people who each know someone who is unemployed is always higher than 90% of the population[未提及的内容]

(D) Roland is not consciously distorting the statistics he presents
            
B

(E) knowledge that a personal acquaintance is unemployed generates more fear of losing one's job than does knowledge of unemployment statistics [无关选项]

 

139.

Sharon’s argument assumes that people are generally similar in how likely they are to have among their acquaintances people who are unemployed. Since heavy concentrations of unemployment in geographically isolated segments of the population would produce great differences in this respect, Sharon’s argument assumes few, if any, such concentrations. Choice B is therefore the best answer.

 

If normal levels of unemployment were exceeded relatively frequently, and if Roland’s figure of 90 percent were an exaggeration, Sharon’s argument would be unaffected, so choices A and D are incorrect. At exceptionally low levels of unemployment, Sharon’s argument suggests that choice C is likely to be false, so C is not assumed. The fear of losing one’s job is not part of Sharon’s argument, so choice E is incorrect.

 

151. Generally scientists enter their field with the goal of doing important new research and accept as their colleagues those with similar motivation. Therefore, when any scientist wins renown as an expounder of science to general audiences, most other scientists conclude that this popularizer should no longer be regarded as a true colleague.

Aè科学家都追求新的重要研究

Bè因此,如果一个学者被大众广泛接受,则不再被认为是研究者

Cè如果被大众广泛接受的,就不是with motivation”

[前提为A,结论为B,假定对结论进行解释]

The explanation offered above for the low esteem in which scientific popularizers are held by research scientists assumes that

(A) serious scientific research is not a solitary activity, but relies on active cooperation among a group of colleagues

(B) research scientists tend not to regard as colleagues those scientists whose renown they envy

(C) a scientist can become a famous popularizer without having completed any important research

(D) research scientists believe that those who are well known as popularizers of science are not motivated to do important new research
        
D

(E) no important new research can be accessible to or accurately assessed by those who are not themselves scientists
            

 

151.

The passage indicates that research scientists accept as colleagues only scientists with motivation to do important new research. This fact explains the tendency of scientists to reject scientists who are renowned popularizers of science only if research scientists believe popularizers lack such motivation; choice D is the best answer.

 

Since the passage is concerned only with whether certain scientists have the goal of doing important new research, not with how research is done, or with who understands new research, choices A and E are both incorrect. Choice B is incorrect because it suggest an alternative explanation of rejection of popularizers. Since the explanation offered remains unaffected even if unsuccessful research scientists cannot become famous popularizers, choice C is incorrect

5#
 楼主| 发表于 2007-8-25 16:01:00 | 只看该作者

162题

 

162. The interview is an essential part of a successful hiring program because, with it, job applicants who have personalities that are unsuited to the requirements of the job will be eliminated from consideration.

Aè通过面试,哪些不符合工作需求的人将被排除

Bè面试是成功招聘的程序

Cè面试总能正确辨别

[前提为A,结论为B,假定对原因失败的可能进行排除]

 

The argument above logically depends on which of the following assumptions?

(A) A hiring program will be successful if it includes interviews.[无关选项]

(B) The interview is a more important part of a successful hiring program than is the development of a job description. .[未提及的比较]

(C) Interviewers can accurately identify applicants whose personalities are unsuited to the requirements of the job.
            

(D) The only purpose of an interview is to evaluate whether job applicants’ personalities are suited to the requirements of the job. .[未提及] C

(E) the fit of job applicants’ personalities to the requirements of the job was once the most important factor in making hiring decisions.
            
.[
未提及]

 

162.

If interviewers cannot accurately identify unsuitable applicants, the interviews cannot play the role that is claimed to make them an essential part of a successful hiring program. Thus the argument depends on choice C being true, making C the best answer.

 

Although the argument claims that the interview is an essential part of a successful hiring program, the interview need not ensure success (contrary to choice A), nor need it b e more important than another part (contrary to choice B). The interview can also have other purposes, such checking on technical qualifications, so D is not depended upon. Nothing is implied about how past hiring decisions were made, so there is no dependence on choice E either.

6#
 楼主| 发表于 2007-8-25 16:01:00 | 只看该作者

167

167. Because no employee wants to be associated with bad news in the eyes of a superior, information about serious problems at lower levels is progressively softened and distorted as it goes up each step in the management hierarchy. The chief executive is, therefore, less well informed about problems at lower levels than are his or her subordinates at those levels.

Aè雇员不希望在上司眼里与不好的消息关联,底层的严重问题通常被逐步扭曲

Bè首席执行官对于底层的信息了解得比他直接下属的问题少

Cè没有其它渠道可以让chief excutive了解基层的情况

[前提为A,结论为B,假定弥补Gap]

 

The conclusion drawn above is based on the assumption that

(A) problems should be solved at the level in the management hierarchy at which they occur[无关选项]

(B) employees should be rewarded for accurately reporting problems to their superiors[正向改善,不是假设无关选项]

(C) problem-solving ability is more important at higher levels than it is at lower levels of the management hierarchy[无关选项]

(D) chief executives obtain information about problems at lower levels from no source other than their subordinates D

(E) some employees are more concerned about truth than about the way they are perceived by their superiors
            
[
无关选项]

 

167.

Unless chief executives rely solely on their subordinates for information about problems at lower levels, the progressive softening and distorting of information described in the passage need not bar the chief executive from obtaining accurate information. Thus, the conclusion that the chief executive is comparatively poorly informed about such problems is based on assuming choice D, which is therefore the best answer.

 

None of the other choices is assumed. Choices A and B are recommendations that the facts in the passage might support. The issue of where problem-solving ability is best deployed (choice C) may be affected by the conclusion’s truth or falsity, but need not be decided in order to draw the conclusion. Choice E, if true, would tend to counteract the phenomenon the passage describes.

7#
 楼主| 发表于 2007-8-25 16:02:00 | 只看该作者

继续

 

171. A recent report determined that although only three percent of drivers on Maryland highways equipped their vehicles with radar detectors, thirty-three percent of all vehicles ticketed for exceeding the speed limit were equipped with them. Clearly, drivers who equip their vehicles with radar detectors are more likely to exceed the speed limit regularly than are drivers who do not.

Aè3%的车安装了雷达”33%的所有被罚款的车安装了雷达è
        
不清楚33%的数据是否具有代表性è孤立数据

Bè因此,安装雷达者比没有安装的更倾向于超速è
        
普遍性结论,讨论的是频率

Cè安装了雷达的车比没有安装的更经常超速è弥补前提的数值与结论的频率间的GAP

 

这题用mindfreegap法基本就可以解决了。我再引用OG的补充一点:结论的重点是regularly经常性超速,而题干的数据是孤立的数据。这题就是典型的由偶然孤立的数据---》有代表性的经常性数据。之间的GAP就是,偶然的数据有代表性。

 

[前提为A,结论为B,假定弥补Gap]

The conclusion drawn above depends on which of the following assumptions?

(A) Drivers who equip their vehicles with radar detectors are less likely to be ticketed for exceeding the speed limit than are drivers who do not.

(B) Drivers who are ticketed for exceeding the speed limit are more likely to exceed the speed limit regularly than are drivers who are not ticketed.

(C) The number of vehicles that were ticketed for exceeding the speed limit was greater than the number of vehicles that were equipped with radar detectors.

(D) Many of the vehicles that were ticketed for exceeding the speed limit were ticketed more than once in the time period covered by the report. B

(E) Drivers on Maryland highways exceeded the speed limit more often than did drivers on other state highways not covered in the report.

 

171.

The conclusion concerns regularly exceeding the speed limit, but the data derive from isolated occasions when drivers exceed the speed limit and are ticketed. The conclusion thus assumes that these instances provide evidence of regular behavior-that drivers ticketed for exceeding the speed limit are likely to be drivers who regularly exceed it. Choice B states this assumption and is the best answer.

 

Choices A, C, and D provide additional data that might be relevant to the conclusion, but if choice B is assumed, the additional data are unnecessary for drawing the conclusion. The difference that choice E describes between Maryland and other states would simply suggest that the report’s findings cannot be extrapolated to other states. It does not help in drawing the conclusion.

 

173. Researchers have found that when very overweight people, who tend to have relatively low metabolic rates, lose weight primarily through dieting, their metabolisms generally remain unchanged. They will thus burn significantly fewer calories at the new weight than do people whose weight is normally at that level. Such newly thin persons will, therefore, ultimately regain weight until their body size again matches their metabolic rate.

Aè研究发现偏重的人拥有偏低的代谢率,通过减食方式减肥,但是代谢会调整到燃烧更少脂肪

Bè因此这些新近减肥的人最终会回复到之前的体重直到与其新的代谢率吻合

Cè很少人能一直维持低卡路里的饮食

[前提为A,结论为B,假定弥补Gap]

 

The conclusion of the argument above depends on which of the following assumptions?

(A) Relatively few very overweight people who have dieted down to a new weight tend to continue to consume substantially fewer calories than do people whose normal weight is at that level.

(B) The metabolisms of people who are usually not overweight are much more able to vary than the metabolisms of people who have been very overweight.

(C) The amount of calories that a person usually burns in a day is determined more by the amount that is consumed that day than by the current weight of the individual.

(D) Researchers have not yet determined whether the metabolic rates of formerly very overweight individuals can be accelerated by means of chemical agents.
            
A

(E) Because of the constancy of their metabolic rates, people who are at their usual weight normally have as much difficulty gaining weight as they do losing it.
                

 

173.

If compared with people who have not been overweight, newly thin people burned fewer calories but also generally consumed fewer calories, one would not reliably conclude that the newly thin people would regain weight. Therefore, the conclusion assumes that the newly thin do not generally consume few calories, making choice A the best answer.

 

The conclusion does not rely on differences in the variability of the metabolism (choice B), just on differences in the rate of metabolism, nor does it rely on the relative significance of different factors in determining how many calories a person burns in a day (choice C). Neither does the conclusion assume anything about whether accelerators for the metabolism have been discovered (choice D), or about why some people have difficulty gaining weight (choice E).

 

195. A proposed change to federal income tax laws would eliminate deductions from taxable income for donations a taxpayer has made to charitable and educational institutions. If this change were adopted, wealthy individuals would no longer be permitted such deductions. Therefore, many charitable and educational institutions would have to reduce services, and some would have to close their doors.

Aè税收政策会取消对那些用于慈善用途的收入的减免,富人们将不再享有哪些减免

Bè因此很多慈善和教育机构将倒闭

Cè
        
慈善和教育机构没有其它收入来源富人们将因此而减少捐献

[前提为A,结论为B,假定弥补GAP]

 

The argument above assumes which of the following?

(A) Without the incentives offered by federal income tax laws, at least some wealthy individuals would not donate as much money to charitable and educational institutions as they otherwise would have.

(B) Money contributed by individuals who make their donations because of provisions in the federal tax laws provides the only source of funding for many charitable and educational institutions.

(C) The primary reason for not adopting the proposed change in the federal income tax laws cited above is to protect wealthy individuals from having to pay higher taxes.

(D) Wealthy individuals who donate money to charitable and educational institutions are the only individuals who donate money to such institutions.
            
A

(E) Income tax laws should be changed to make donations to charitable and educational institutions the only permissible deductions from taxable income.
                

 

195.

The passage argues that charitable and educational institutions, part of whose income comes from donations, would be negatively affected if wealthy individuals could not count such donations as deductions from their income. The question asks you to identify an assumption of the argument-that is, something that has to be true in order for the evidence presented to establish the conclusion.

 

Choice A is the best answer, since if this statement is false, all wealthy individuals would, even without the incentive provided by federal tax laws, donate as much money as they do now. In that case, the evidence used in the argument provides no support for the conclusion.

 

Choice B is not assumed: the argument need only assume that many institutions depend heavily, but not necessarily exclusively, on donations from such individuals. Choice C is incorrect given that the argument is concerned only with the consequences of the proposed change and makes no assumption about any reasons for making or not making the change. Choice D is not assumed: as far as the argument is concerned, there can be many other individuals who donate money to the institutions. Choice E is incorrect since the argument, being about the consequences of the particular proposed change, does not make any assumption about what alternative changes to the tax laws ought to be made.

 

202.Gortland has long been narrowly self-sufficient in both grain and meat. However, as per capita income in Gortland has risen toward the world average, per capita consumption of meat has also risen toward the world average, and it takes several pounds of grain to produce one pound of meat. Therefore, since per capita income continues to rise, whereas domestic grain production will not increase, Gortland will soon have to import either grain or meat or both.

Aè在肉食和谷物方面G一直自给自足,由于收入上涨,肉的消费量也上涨,而每磅肉需要几磅谷物饲养

Bè因此G将不得不进口谷物/肉食或者两者都进口

Cè “G不会因为肉食消费上涨就减少谷物的消费量

[前提为A,结论为B,假定弥补GAP]

 

Which of the following is an assumption on which the argument depends?

(A) The total acreage devoted to grain production in Gortland will not decrease substantially.

(B) The population of Gortland has remained relatively constant during the country’s years of growing prosperity.

(C) The per capita consumption of meat in Gortland is roughly the same across all income levels.

(D) In Gortland, neither meat nor grain is subject to government price controls. E

(E) People in Gortland who increase their consumption of meat will not radically decrea
        
se their consumption of grain.
            

 

202.

The argument in the passage concludes that, although Gortland currently produces enough grain and meat for its own needs, it will soon not do so. This conclusion is based on the continuing increase in per capita consumption of meat as per capita income increases, and the fact that several pounds of grain must be used to produce each pound of meat.

 

The question asks you to identify an assumption on which the argument depends. An assumption is something that must be true in order for the argument’s conclusion to be established by the evidence the argument gives.

 

Choice E is the best answer. If the people who increase their consumption of meat at the same time radically reduce their consumption of grain, the evidence given in the argument cannot establish its conclusion. So for the conclusion to be established this possibility must be ruled out, which is what this answer choice does.

您需要登录后才可以回帖 登录 | 立即注册

Mark一下! 看一下! 顶楼主! 感谢分享! 快速回复:

手机版|ChaseDream|GMT+8, 2024-9-29 20:18
京公网安备11010202008513号 京ICP证101109号 京ICP备12012021号

ChaseDream 论坛

© 2003-2023 ChaseDream.com. All Rights Reserved.

返回顶部